Sunteți pe pagina 1din 68

[Type text]

COMPETENCY
APPRAISAL
STUDENT GUIDE























DISCLAIMER

Medical knowledge is constantly changing. Standard safety precautions must
be followed, but as new research and clinical experience broaden our
knowledge, changes in treatment and drug therapy may become necessary
or appropriate. It is extremely difficult to ensure that all information
presented is entirely accurate for all circumstances and the author and
contributors cannot accept responsibility for any errors and omissions. The
authors and contributors make no warranty, express or implied, with respect
to this work, and disclaim any liability, loss, or damage as a consequence,
directly or indirectly of the use and application of any of the contents of this
work.



























3










I fall too fast,
I crash too hard,
I forgive too easy,
And care too much.

For my crazy brothers,
my cheerful friends,
my loving mom and aunts,
my sweet honey
and for the most special person in my life
Him






















TABLE OF CONTENTS

Cover Page..
Disclaimer.
Dedication.
Table of Contents..
Foreword
Acknowledgements.
Foundations in Nursing .
Fundamentals of Nursing
Math for Nurses .
Community Health Nursing
Communicable Diseases ..
Maternal and Child Nursing ..
Medical-Surgical Nursing
Disorders of oxygenation
Disorders of metabolism
Disorders of elimination
Disorders of perception
Research ..
Answers and rationales .
References.

















5


FOREWORD

Competency Appraisal deals with the application of the concepts, principles
and processes basic to the practice of nursing with emphasis on health
promotion, disease prevention and risk reduction, health maintenance,
curative and rehabilitative aspects of care of sick mother, child and family,
population group and community, sick individuals with alterations in
oxygenation, fluid and electrolyte balance, metabolism, perception and
coordination, cellular aberrations, adjustment problems, and maladaptive
patterns of behavior, disaster and emergency. It includes the utilization of
the nursing process and core competencies under the eleven key areas of
responsibility.





























ACKNOWLEDGEMENT

The making of this project is like feeding your brain with sweet candy, it is
nice. A lot of time is consumed. Its goes harder before it goes easier. But it
will only get better; you got to make it through the hard time first. And when
stress gets sour, sweeten it with gratitude.

Each day I am thankful for the nights that turned mornings; friends that
turned to family; dreams that turned to reality; and the likes that turned to
love.

Father God, I want to take a minute not to ask anything from you but simply
thank you for all that I have.

My family, especially my mom whom I have always honored and cherished,
thank you so much for being supportive. You have never failed to make me
realize that life is like a wheel, full of ups and downs. Words will never be
enough to tell you how grateful I am, to be your daughter.

My friends, who where always there to comfort me and boost up my
confidence whenever I feel down; I am so lucky to be a part of your life. I
dont know what Ill do without all of you. Let us continue to influence each
other with good vibes and throaty laughter.

My beloved, we may be miles apart and I know we still have a lot of things to
conquer but my heart will always beat for you. Having you in my life is
already an answered prayer. Thank you for always being there for me.

To all nursing students and future readers of this book, continue learning
and improve your skills because our profession is dynamic. It changes. Like a
metal bar, do not let your skills and wisdom get rusted.









7

FOUNDATIONS OF NURSING
1. During the American Civil War, several nurses emerged and were
noted for their contribution to a country town by internal strife.
Among the nurse leaders, who was considered as the Moses of her
people?
A. Louisa May Alcott
B. Sojourner Truth
C. Harriet Tubman
D. Dorothea Dix
2. Religion played a significant role in the development of nursing.
Several orders of knights provided nursing care to their sick and
injured comrades. These brothers in arms were called?
A. Fabiola
B. The Crusades
C. Teutonic Knights
D. The Knights of Saint Lazarus
3. She was considered the founder of Public Health Nursing.
A. Lilian Wald
B. Lavinia L. Dock
C. Mary Mahoney
D. Margaret Sanger
4. Health promotion is best represented by which of the following
activities?
A. Administering immunization
B. Giving a bath
C. Preventing accidents in the home
D. Performing diagnostic procedures
5. It is a process that engages in activities and behaviors that enhance
quality of life and maximize personal potential?
A. Rehabilitation
B. Prevention
C. Hospice Care
D. Wellness
6. Nurses assume a number of roles in providing quality care to their
clients. The following are the roles and functions of the nurse
except:
A. Communicator
B. Nurse Administrator
C. Change Agent
D. Nurse Manager
7. Which of the following is an example of continuing education for
nurses?
A. Completing a workshop on ethical aspects of nursing
B. Obtaining information about the facilitys new computer
charting system
C. Attending the hospitals orientation program
D. Talking with a company representative about a new piece
of equipment
8. A client who only speaks and understands their own native tongue
should be cared by:
A. Speaking loudly and slowly
B. Standing close to the client and speaking loudly
C. Arranging for an interpreter when communicating
D. Speak to the client and family the chances that the topic
will be understood.
9. Hildegard Peplau is a psychiatric nurse who introduced her
interpersonal concept wherein the center of her theory is the use of
therapeutic relationship. In this phase, the client derives full value
from what the nurse offers through the relationship.
A. Exploitation
B. Resolution
C. Orientation
D. Identification
10. Which of the following provides the best explanation for describing
nursing as a practice discipline?
A. Nursing focuses on performing the professional role
B. It takes an experience to become a competent nurse
C. Research and theory development is a central focus
D. Nurses functions as members of a team who form a
practice group.
11. Persons, environment, health and nursing encompass the
metaparadigm of Nursing because of what reason?
A. Those are the recipients of nursing care
B. Those are the internal and external surroundings that
affect our clients
C. They focus on the needs of our clients
D. They can be utilized in any setting when caring for a client
12. It deals with the relationships among individuals in the society.
A. Criminal actions
B. Civil actions
C. Rights
D. Responsibilities
9

13. The process of determining and maintaining competence in nursing
practice.
A. Litigation
B. Delegation
C. Causation
D. Credentialing
14. Which of the following patients is not allowed to provide consent?
A. A female adult client who has been blind since birth.
B. A 13-year old street junkie who is an addict and is to
receive a treatment for substance abuse.
C. A 40-year old male who is positive for HIV.
D. A physically fit nurse who is diagnosed with Paranoid
Schizophrenia and is incompetent.
15. A crime is an act committed in violation of public (criminal) law and
punishable by a fine or imprisonment, which of the following acts
can be considered as a misdemeanor?
A. A nurse who accidentally gave a lethal dose of narcotic.
B. A midwife who has a birth clinic and is practicing abortion
to unwanted teenage pregnancies.
C. A nursing student who slapped an annoying adolescent in
a Pediatric Ward.
D. A nursing attendant who confronted a patient for having a
positive HIV/AIDS result.


FUNDAMENTALS IN NURSING
16. Nurses are expected to use critical thinking to solve client problems
and make better decisions. Which of the following acts done by a
nurse does not use critical thinking?
A. Using knowledge on nutrition, physiology, and physics in
educating a client with pressure ulcer
B. Offering a frightened child to blow bubbles instead of using
incentive spirometry.
C. Assuming that all child loves peanut butter sandwich
D. Deciding what observations on a client needs to be
reported to the primary health care provider immediately
and those which are to be addressed later.
17. Certain attitudes are crucial in critical thinking. Which of the
following attitudes should be developed by a critical thinker?
A. Curiosity
B. Intuition
C. Seeking alternatives
D. Projecting
18. An 80-year old Asian woman is diagnosed with DM type 1. The
healthcare provider then orders to give insulin injection once CBG
result is abnormal. The nurse collects blood sample after warming
the clients hands. This act done by the nurse is based from a ___?
A. Fact
B. Inference
C. Judgment
D. Opinion
19. Complete database provides a baseline for comparing the clients
responses to nursing and medical interventions. Which of the
following is not a component of a nursing health history?
A. Personal habits
B. Client records
C. Occupational history
D. Communication style
20. If the client is complaining for abdominal pain, what physical
assessment should be done first?
A. Palpate
B. Auscultate
C. Inspect
D. Percuss
21. A 35-year old female went to the out-patient department and is
scheduled for Papanicolaou smear. The nurse is correct in
positioning the client if:
A. The client lies on its abdomen with head turned to the
side.
B. The client lies on its back with legs extended.
C. The client lies on its back with feet supported in stirrups
and hips is in lined with the edge of the table.
D. The client lies on its back with knees flexed and hips
externally rotated with a small pillow under the head and
soles of feet on the surface.
22. A child complains of difficulty in hearing and foul discharge is
draining from its right ear. An otoscopic exam is to be done. The
nurses initial nursing intervention would be:
A. Standing 1-2 feet from the client, covers his/her mouth,
exhales fully and softly whispers two-syllable words on the
unocluded ear of the client.
B. Checking auditory canal for presence of foreign bodies.
11

C. Instructing the client to stay still during the exam.
D. Holding the base of the tuning fork and lightly taps the
lines on the other hand.
23. During physical assessment, the nurse on duty is inspecting the skin
of her client. She noticed a circumscribed, round thin translucent
mass filled with serous fluid. This skin lesion is an example of:
A. Vesicle/bulla
B. Macule/patch
C. Wheal
D. Pustule
24. Which of the following is an expected finding during the assessment
of the older adult?
A. Facial hair becomes finer and softer.
B. Decreased peripheral, color and night vision.
C. Increased sensitivity to odors.
D. Respiratory rate and rhythm are irregular at rest
25. If client reports loss of short-term memory, the nurse would assess
using which of the following?
A. Have the client repeat series of 3 numbers, increasing to
eight if possible.
B. Ask the client to count backwards from subtracting seven
each time.
C. Ask the client to describe how he/she arrived at the clinic.
D. Have the client describe how often he gets sick during his
childhood.
26. While performing a cardiac assessment on a client with
incompetent heart valve, the nurse auscultates a murmur. Which of
the following best describes the sound of a heart murmur?
A. Lub-dub sound
B. Scratchy, leathery heart noise
C. Gentle, blowing or swooshing noise
D. Abrupt, high-pitched snapping noise
27. Self-breast examination is done to palpate for masses, tenderness
or any discharge from the nipples. The following data are to be
recorded if mass is located except:
A. Location
B. Nipple
C. Perfusion
D. Consistency
28. In assessing the genitals and inguinal areas of the client, which of
the following findings is not a normal finding with regards to
children?
A. Testicles ascend in the inguinal canal after a cremasteric
reflex
B. Penis increases in both breadth and length
C. Pubic hair is present in preadolescence
D. Epididymis is resilient, normally tender and softer than
spermatic cord
29. If unable to locate the clients popliteal pulse during a routine
examination, the nurse should perform which of the following next?
A. Check for a pedal pulse
B. Check for the femoral pulse
C. Take the clients blood pressure on the thigh
D. Ask another nurse to try to locate the pulse
30. Which of the following statements is true regarding health
examination?
A. Health examination may entail partial inflammation of the
clients body system assessment.
B. Health assessment is conducted in a systematic manner
which requires frequent change in position.
C. Data obtained in physical examination will only confirm the
data obtained during the nursing history.
D. Initial assessment findings provide baseline data about
clients functional abilities.


MATH FOR NURSES
31. The physician ordered potassium penicillin 500,000 U q4h, IVPB.
Nurse Happy is correct if she interprets this as:
A. A potassium penicillin 500,000 unit is to be injected by vein
every 4 hours.
B. Potassium penicillin is to be administered via intravenous
route for 4 hours.
C. Administer 500,000 units potassium penicillin by
intravenous route four times every hour.
D. Administer potassium penicillin 500,000 units by
intravenous piggyback every 4 hours
32. A legal drug order must indicate:
A. Patients full name, name of drug, name of physician
B. Patients full name, date of the order, name of medication,
physicians name
13

C. Patients full name, date of the order, name of medication,
dose, frequency and route of administration
D. Patients full name, name of medication, date of the order,
dose, frequency and route of administration, physicians
name
33. A patient is to receive allopurinol (Zyloprim) 0.2g daily by mouth.
Zyloprim is available in 100 mg tablets. How many tablets will the
patient receive?
A. 4 tablets
B. 2 tablets
C. 1 tablet
D. 1 tablets
34. A patient is to receive potassium chloride 30 mEq 10% oral solution.
The label on the container reads: Potassium Chloride 20 mEq per 15
mL 10% oral solution. How many mL should the nurse administer?
A. 22. 5 mL
B. 23 mL
C. 21 mL
D. 21.5 mL
35. A patient is to receive fentanl citrate (Sublimaze) 0.4mg IM.
Available is a multidose vial labeled Sublimaze 50g/mL. How much
of the drug should the nurse administer?
A. 0.8 mL
B. 8 mL
C. 0.08 mL
D. 80 mL
36. Mila, a DM patient, is to receive Novolin 70/30 insulin 76 units SC
by injection. The multidose vial of Novolin 70/30 insulin is labeled
100 units/mL (10mL). The nurse using an insulin syringe should
administer how many units; or how many mL using a tuberculin
syringe?
A. 76 units, 7.6 mL
B. 0.76 mL, 76 units
C. 7.6 mL, 760 units
D. 7.6 units, 7.6 mL
37. The physician has prescribed methotrexate (Methotrexate LPF) 39
mg once a week as maintenance dose for a 12-year old who weighs
40 kg and is 150 cm tall. The manufacturers recommended average
dose is 30mg/m
2
, BSA is 1.3m
2
. The nurse will administer how many
mL if the drug label reads 250mg/10mL (25mg/mL)?
A. 16 mL
B. 0.16 mL
C. 1.6 mL
D. 1.06 mL
38. Infuse Velosef 1g in 100mL solution intravenously over a 30-minute
period. The drop factor is 10gtts/mL.
A. 19 gtts/min
B. 30 gtts/min
C. 25 gtts/min
D. 33 gtts/min
39. Determine the flow rate of a patient who is 154 lbs with a
prescribed medication of 0.5mg/kg/min. The medicine available is
2.0g per 100mL and the drop factor is 60gtts/mL.
A. 100 gtts/min
B. 105 gtts/min
C. 200 gtts/min
D. 205 gtts/min
40. The physicians order states: Add 25 mEq of potassium chloride to
1000mL D
5
W. Available are ampoules labeled potassium chloride
40mEq = 20mL. How much should be added?
A. 125mL
B. 1.25mL
C. 12.5mL
D. 1,250mL
41. The physician has ordered a prediluted solution of theophylline 500
mg in 250mL D
5
W to be infused at a rate of 750g per minute for
the patient. The IV administration set delivers 60gtts/mL. How
many mL/hr will deliver this dosage?
A. 225mL
B. 2, 250mL
C. 22.5mL
D. 2.25mL


COMMUNITY HEALTH NURSING
42. Which of the following is not true regarding the community
organizing and participatory action research?
A. It is a tool helping community health workers to generate
community participation and obtain information for
developing a community plan for health as a key to
development.
15

B. Participatory action research is used to intervene and
initiate change and development in the families,
population groups and the entire community.
C. Action research involves utilizing a noncyclical method.
D. One of the function of participatory action research is to
study the impact and effect of the nurses delectations or
intervention in the family/community.
43. Also refers as agitation, this entails going around and motivating
people on the one-to-one basis to do something about community
issues.
A. Groundwork
B. Social investigation
C. Mobilization
D. None of these
44. The lowest level of reporting unit where it is expected to report
health services provided to its defined catchment area is:
A. Barangay Health Section
B. Rural Health Unit
C. District Hospital
D. Provincial Hospital
45. Which of the following major aspects of community health
paradigm is focused on changing and modifying health practices by
increasing and developing knowledge, attitudes and skills?
A. Health care knowledge
B. Health education
C. Health care practices
D. All of the above
46. Newborn Screening Act of 2004 is an act promulgating a
comprehensive policy and a national system for ensuring newborn
screening. This act is also known as:
A. RA 9288
B. RA 8976
C. RA 8749
D. RA 8172
47. This law aims to transform the local government unit into self-
reliant communities and active partners in the attainment of
national goals although a more responsive and accountable local
government structure instituted through a system of
decentralization?
A. RA 7305
B. RA 7160
C. RA 7875
D. RA 7432
48. Which is not a principle of community organizing?
A. Basic trust on the people is essential in community
organizing work
B. Imposed changes have more meaning and permanence
than self-willed changes
C. Poor and the oppressed are open to change
D. Social awareness is needed for people to change their
situation
49. What is the overall goal of Family Planning?
A. Focus service delivery to the urban and rural poor
B. Re-establish the family planning outreach programs
C. Strengthen family planning provision in regions the high
unmet needs
D. Provide universal access to family planning information
and services whenever and wherever needed.
50. A critical step in building peoples organization that involves area
selection and acquiring demographic and geographic profile:
A. Entry
B. Preparatory
C. Integration
D. Organizational
51. During clinic visit, which of the following is implemented during pre-
consultation conference?
A. Assist client before, during and after the examination by
the physician
B. Ensure privacy, safety and comfort of the patient
C. Take clinical history
D. Observe confidentiality of examination results
52. Which of the following best describes empowerment?
A. A process by which individuals, communities and
organizations gain mastery over their lives
B. When various members of the community care able to
collaborate effectively on identifying the problem and
needs of the community
C. The ability of the community is to engage in effective
problem solving
D. Refers to the need of the community members to be active
rather than passive in the learning process
17

53. As a nurse lecturer of community health nursing, you would stress
that supervision includes which of the following except:
A. Supervision is done to develop the capability of the worker
to solve own problems and meet own need by providing
continuing personal guidance and professional
development.
B. Most of the supervision functions are directed to the
higher level of workers.
C. Supervision is not based on set of rules or formula but on
each supervisory situation.
D. Supervision is considered as a facilitating process that
consists of inspecting and evaluating the work of another
to remedy rather than punish flood performance.
54. Community health nurses utilize not only community organizing but
also participatory action research to intervene and initiate change
and development in the community. Which of the following is not a
function of PAR?
A. Obtain data assessment and diagnosis of the community.
B. Preparation of community nursing care plan.
C. PAR is used to study the impact and affect nurses direct
actions/interventions in the family/community.
D. In involves utilizing a systemic non-cyclical method of
planning, taking action, observing, evaluating and critical
reflecting.

COMMUNICABLE DISEASES
55. The nurse is caring for a client with acquired immunodeficiency
syndrome (AIDS). To adhere to standard precautions, the nurse
should:
A. Maintain strict isolation
B. Keep the client in a private room if possible
C. Wear gloves when providing mouth care
D. Wear a gown when delivering the clients food tray
56. The nurse encourages the client with an immunologic disorder to
eat a nutritional balanced diet to promote optimal immunologic
function. Autoimmunity has been linked to excessive ingestion of:
A. Protein
B. Fat
C. Vitamin A
D. Zinc
57. A client with Herpes Zoster is prescribed acyclovir (Zovirax) 200mg
PO every 4 hours while awake. The nurse should inform the client
that this drug may cause:
A. Palpitations
B. Dizziness
C. Diarrhea
D. Metallic taste
58. After diagnosing a client with pulmonary tuberculosis, the physican
tells the family members that they must receive isoniazid (Laniazid)
as prophylaxis against tuberculosis. A clients teenage daughter asks
the nurse how long the drug must be taken. What is the usual
duration of the prophylactic isoniazid therapy?
A. 3-5 days
B. 1-3 weeks
C. 2-4 months
D. 6-12 months
59. A client of pneumocystis carinii pneumonia is receiving
trimetrexate. The rationale for administering leucoverin calcium to
a client receiving methotrexate is to:
A. Treat anemia
B. Create a synergistic effect
C. Increase the number of white blood cells
D. Reverse drug toxicity
60. The principal mosquito vector in Filariasis is:
A. Aedes poecilus
B. Mansonia bonnea
C. Anopheles flavivoris
D. Mansonia uniformis
61. Varicella zoster virus is the etiologic agent for which of the
following diseases?
A. Mumps
B. Diphtheria
C. Pertussis
D. Chicken pox
62. Which of the following behaviors indicate that a client understands
measures that may prevent her from acquiring influenza?
A. The client covers nose and mouth when sneezing or
coughing.
B. The client routinely takes prophylactic antibiotics
C. The client receives the appropriate flu vaccine each year.
D. The client asks to have a throat culture done to detect
infection.
19

63. Which of the following terms describes a fungal infection found on
the upper arm?
A. Tinea capitis
B. Tinea corporis
C. Tinea cruris
D. Tinea pedis
64. A 7-year old child is diagnosed with head lice. The mother asks how
to get rid of the nits. The nits represent which of the following life
cycle of the louse?
A. Adult
B. Empty egg shells
C. Newly laid eggs
D. Nymph


MATERNAL AND CHILD NURSING
65. A nurse is caring for a post-partum client, which finding would make
the nurse suspect endometritis in this client?
A. Lochia rubra on the second day of postpartum
B. Fever over 38C, beginning three day post partum
C. Elevated WBC
D. Breast engorgement
66. A client see in the health care clinic has been diagnosed with
endometriosis and asks the nurse to describe this condition the
nurse tells the client that endometriosis:
A. Is the presence of tissue outside the uterus that resembles
the endometrium
B. Is pain that occurs during ovulation
C. Is also known as primary dysmenorrhea
D. Causes the cessation of menstruation
67. A nurse is assessing a client with cardiac disease at the 30-week
gestation antenatal visit. The nurse assesses lung sounds in the
lower lobes following a routine blood pressure screening. The nurse
performs this assessment to:
A. Identify cardiac dysrhythmias
B. Rule out the possibility of pneumonia
C. Assess for early signs of CHF
D. Identify mitral valve prolapsed
68. A woman at 32 weeks gestation is brought into the ER after an
automobile accident. The client is bleeding vaginally and fetal
assessment indicates moderate fetal distress. Which of the
following will the nurse do first in an attempt to reduce the stress
on the fetus?
A. Start intravenous (IV) fluids at a keep open rate
B. Administer oxygen via face mask at 7-10 LPM
C. Elevate the head of bed to a semi-fowlers position
D. Set up for an immediate CS delivery
69. A nurse is caring for a pregnant client with a history of HIV. Which
nursing diagnosis formulated by the nurse has the highest priority
for this client?
A. Self care deficit
B. Risk for infection
C. Imbalanced nutrition
D. Activity intolerance
70. Nurse Mark is assessing a three-day old preterm neonate with
diagnosis of respiratory distress syndrome. Which assessment
finding indicates that the neonates respiratory status is improving?
A. Presence of systolic murmur
B. Respiratory rate between 60-70
C. Edema of the hands and feet
D. Urine output of 1-3ml/kg/hr
71. A 3 month-old infant just had a cleft lip and palate repair. To
prevent trauma to the operative site, the nurse should do which of
the following?
A. Give the baby a pacifier to help soothe him.
B. Lay the baby in the prone position.
C. Place the infants arms in soft elbow restraints.
D. Avoid touching the suture line, even to clean.
72. A childbirth educator tells a class of expectant parents that it is
standard routine to instill a medication into the eyes of a newborn
infant as a preventive measure against ophthalmia neonatorum.
The educator tells the class that the medication used for
prophylaxis of ophthalmia neonatorum is:
A. Erythromycin ophthalmic eye ointment
B. Neomycin ophthalmic eye ointment
C. Penicillin ophthalmic eye ointment
D. Vitamin K injection
73. A 5 year old child is hospitalized with Rocky Mountain Spotted
Fever. The nursing assessment reveals that the child was bitten by a
tick 2 weeks ago. The child presents with complains of headache,
fever, and anorexia, and the nurse notes a rash on the palms of the
hands and soles of the feet. The nurse reviews the physicians
21

orders and anticipates that which of the following medications will
be prescribed?
A. tetracycline (Achromycin)
B. amphothericin B (Ketoconazole)
C. ganciclovir (Foscamet)
D. amantadine (Rimantadine)
74. A nurse teaches a mother of a newly circumcised infant about post
circumcision care. Which statement by the mother indicates
understanding of the care required?
A. I need to check for bleeding every hour for the first 12
hours.
B. I need to clean the penis every hour with baby wipes.
C. I need to wrap the penis completely in dry sterile gauze,
making sure it is dry when I change his diaper.
D. My baby will not urinate for the next 24 hours because of
the swelling.
75. A mother brings her 2 week-old infant to a clinic for treatment
following a diagnosis of clubfoot made at birth. Which statement by
the mother indicates a need for further teaching regarding this
disorder?
A. Treatment needs to be started as soon as possible.
B. I realized my infant will require follow-up care until full
grown.
C. I need to bring my infant back to the clinic in 1 month for
a new cast.
D. I need to come to the clinic every week with my infant for
the casting.


MEDICAL-SURGICAL NURSING
DISORDERS IN OXYGENATION
76. An adult client is to receive a unit of whole blood. The clients vital
signs before starting the transfusion are BP- 120/70mmHg, PR-
80bpm, and T- 98.4 F. Five minutes after the transfusion was
started the vital signs are: BP- 100/70mmHg, PR- 100bpm, 99.4F.
What should the nurse do initially?
A. Slow down the rate of transfusion reassess the client in 15
minutes.
B. Stop the transfusion, keep vein open with normal saline
C. Slow down the infusion, notify the physician immediately
D. Administer acetaminophen (Tylenol), continue to monitor
closely throughout the transfusion
77. When assessing a client with angina, which of the following is
considered a common precipitating factor for pain?
A. Exposure to warmth
B. Smoking
C. Prolonged rest
D. Eating a light meal
78. After being diagnosed with an acute MI and stabilized, a female
client denies pain. Her vital signs and heart rhythm are stable but
she appears agitated and uses her call light more often. Which of
the following actions would be the initial step in addressing the
clients needs?
A. Assess her blood pressure frequently.
B. Explain that another episode is unlikely.
C. Encourage her to discuss her feelings about the MI.
D. Tell the client not to worry, she is being closely monitored.
79. Which of the following behaviors indicate that a client diagnosed
with CHF is being compliant with the discharge teaching?
A. Demonstrating better nutrition habits by gaining weight.
B. Returning to the hospital as an in-patient less frequently.
C. Significantly improving his activity level.
D. Attending all the medication teaching classes.
80. The discharged teaching for the client diagnosed with CAD should
include which of the following?
A. Decreasing intake of dietary fiber.
B. Exercising 2 hours before eating.
C. Decreasing cigarette smoking.
D. Participating in regular exercise.
81. Assessment of a client diagnosed with CHF reveals moderate
dyspnea, clammy and very pale skin, and cough producing frothy,
blood-tinged sputum. Based on these findings, the nurse suspects
that the client is experiencing which of the following?
A. Angina
B. Myocardial infarction
C. Pulmonary edema
D. Endocarditis
82. A client is experiencing a cardiac arrest, which of the following
interventions should be performed first?
A. Assess clients blood pressure.
B. Establish a patent airway.
C. Auscultate heart sounds.
23

D. Assist with defibrillation.
83. When assessing a clients radial pulses, the nurse finds them to be
irregular, with the apical pulse rate about 10 beats per minute than
the radial pulse rate. The nurse would suspect which of the
following cardiac arrhythmias?
A. Atrial fibrillation
B. Second-degree AV block
C. Ventricular tachycardia
D. Sinus bradycardia
84. While performing discharge teaching for a client with chronic CHF,
the nurse should stress which of the following topics?
A. Need for high-impact aerobic exercise program.
B. Use of high sodium and low potassium foods.
C. Signs and symptoms of pulmonary edema.
D. Possibility of the need for surgical procedures.
85. After right femoral angiography that occurred 4 hours earlier, the
nurse notices that the clients right leg and foot are cool and pale
and he is unable to palpate any pulses in the foot. Which of the
following nursing actions would the nurse do first?
A. Reassure client that this is a common complication.
B. Notify the clients physician immediately with the findings.
C. Elevate the leg and administer an analgesic as ordered.
D. Ambulate the client to restore circulation to the foot.
86. Which of the following clinical manifestations would the nurse
expect to assess in a client diagnosed with pericarditis?
A. Sharp, sudden pain over the precordium, radiating to the
left scapular region
B. Weakness, fatigue and petechiae of the anterior trunk and
conjunctiva.
C. Crushing chest pain radiating down left arm plus nausea
and vomiting.
D. Hepatomegaly, jugular vein distention and dependent
peripheral edema.
87. A middle-aged client with CAD has been hospitalized three times in
the last 6 months, suggesting non-compliance with the medication
regimen. When preparing the client for discharge this time, which
of the following is the nurses best action?
A. Re-educate the client about the medication and the dosing
schedule.
B. Collect more data to help identify reasons for non-
compliance.
C. Teach the family about the medication and need for
compliance.
D. Arrange for out-patient follow-up examinations to ensure
compliance.
88. During assessment of a client with calf leg pain associated with
exercise, which of the following areas is the priority focus?
A. Leg color, temperature, pulses, pain description
B. Smoking history, DVT history, presence of edema.
C. Allergy and medication history, usual activity pattern.
D. Menopausal state, occupation and socioeconomic status.
89. Which of the following identifies the primary rationale for placing a
clients legs in a dependent position in relation to the heart for a
client with chronic occlusive disease of the lower extremities?
A. Improve activity intolerance.
B. Decrease pain.
C. Reduce risk for disuse syndrome.
D. Improve peripheral blood flow.
90. The client who experiences angina has been told to follow a low-
cholesterol diet. Which of the following meals should the nurse tell
the client would be best on her low-cholesterol diet?
A. Hamburger, salad, and milkshake
B. Baked liver, green beans, and coffee
C. Spaghetti with tomato sauce, salad, and coffee
D. Fried chicken, green beans and skin milk


DISORDERS OF METABOLISM
91. Which of the following nursing interventions would be included in
the plan of care for a client with acute diverticulitis?
A. Administering bulk laxatives and increasing fluid intake.
B. Encouraging high fiber diet and inserting rectal tube.
C. Keeping the client NPO and initiating nasogastric
suctioning.
D. Administering anti-diarrheal medications and encouraging
low-fiber diet.
92. Which of the following would the nurse expect to find when
assessing the client with esophagitis?
A. Mid-epigastric pain and tenderness
B. Abdominal distention and fever
C. Abdominal cramping and vomiting
25

D. Heartburn and dysphagia
93. When developing a teaching plan for a client with GERD, the nurse
should include which of the following instructions?
A. Elevate the foot of the bed
B. Lie down immediately after a meal
C. Take antidiarrheal medication after each loose stool
D. Avoid caffeine, tobacco and peppermint
94. A 17-year old client with a temperature of 100.4C comes into the
emergency room complaining of severe abdominal pain in the RLQ
and has had nausea and vomiting for the last 6 hours. Which of the
following would the nurse suspect?
A. Diverticulitis
B. Appendicitis
C. Gastroenteritis
D. Irritable bowel syndrome
95. Which of the following interventions should the nurse include in
teaching a client having a corticosteroid therapy?
A. Avoid going out in the sun without using a sun block.
B. Stop taking medication if moon face or buffalo hump
occur.
C. Take the medication on an empty stomach.
D. Discontinue the medication gradually by tapering the dose.
96. Which of the following assessment findings indicate that lactulose is
effective in decreasing the ammonia level in the client with hepatic
encephalopathy?
A. Passage of two or three soft stool daily.
B. Evidence of watery diarrhea.
C. Daily deterioration in the clients handwriting.
D. Appearance of frothy, foul-smelling stools.
97. Which of the following reasons would the nurse explain placement
of an esophageal tamponade tube in a client who is having a
hemorrhage?
A. Obtain cooperation and reduce fear.
B. Begin teaching for home care.
C. Allow the client to help insert the tube.
D. Maintain the clients level of anxiety and alertness.
98. Which of the following would be most helpful for a client with
chronic pancreatitis?
A. Modifying protein in the diet
B. Encouraging daily exercise
C. Allowing liberal fluid intake
D. Counseling to stop alcohol consumption
99. Which of the following clinical manifestations would the nurse
expect a client diagnosed with acute cholecystitis to exhibit?
A. Nausea, vomiting and anorexia
B. Ecchymosis, petechiae and coffee ground emesis
C. Jaundice, dark urine, and steatorrhea
D. Acute RLQ pain, diarrhea and dehydration
100. A client with jaundice is experiencing pruritus, which of the
following should be included in the plan of care?
A. Keeping the clients fingernails short and smooth.
B. Applying pressure when giving IM injections.
C. Decreasing the clients dietary protein intake.
D. Administering vitamin K subcutaneously.





DISORDERS OF ELIMINATION
101. A client is complaining of dribbling, urgency and an inability to get
to the bathroom before urinating starts. The nurse suspects which
of the following?
A. UTI
B. Renal calculi
C. Acute renal failure
D. Urinary incontinence
102. The client has a chronic renal failure, he/she should manage fluid
intake by:
A. Subtracting the previous days urine output from 500mL
and limiting intake to milk.
B. Adding 500mL to the previous days urine output and
dividing that amount over the next 24 hours.
C. Consuming all of the fluid allowance during the day to
prevent nighttime bladder distention.
D. Weighing himself or herself before each meal and drinking
500mL of fluid four times each day.
103. The nursing diagnosis knowledge deficit related to the need for
teaching to prevent pyelonephritis would least likely apply to which
of the following clients?
A. A 25-year old sexually active man.
27

B. A bedridden elderly client with an indwelling folley
catheter.
C. An 18-month old toddler with a history of vesicoureteral
relux.
D. A 55 year old woman who has been treated for urinary
retention.
104. Clinical manifestations and assessment findings that support a
diagnosis of acute pyelonephritis include:
A. Urinary stress incontinence and abdominal pain.
B. Flank pain, fever and dysuria.
C. Burning in urination and inflamed urinary meatus.
D. Acute, sharp, intermittent pain and anuria.
105. An 18-year male who has had a renal transplant has expressed
frustration about feeling alone in the hospital room. He wishes to
be out of the waiting room visiting with his friends. Which of the
following responses would be the most appropriate?
A. Allow the client to go to the waiting room for 5 minutes
only.
B. Allow the clients friends to come into the room for a short
visit.
C. Allow the client to verbalize feelings of isolation.
D. Discussed activities that the client can do to combat
isolation.
106. Which of the following would be included in the teaching plan for a
client diagnosed with pelvic relaxation disorder?
A. Instruct the client to perform Kegel exercise daily.
B. Advise client to keep diary of menstrual cycle events.
C. Instruct the client on correct way to administer vaginal
cream.
D. Encourage frequent sexual intercourse until treatment is
completed.
107. Which of the following would be the most appropriate when
teaching a client with phimosis how to prevent recurrence?
A. Discussing the ways to prevent a sickle cell crisis.
B. Instructing the client to apply an ice pack to the penis.
C. Encouraging the client to elevate the scrotum.
D. Teaching proper care of an uncircumcised penis.
108. When taking the history of a 62 year-old client complaining of
impotence, which of the following statements would alert the nurse
to a possible underlying cause?
A. I have had diabetes for the last 12 years.
B. I have never had a rectal digital examination.
C. I have been taking aspirin daily for the last 2 years.
D. I have urinary frequency and terminal dribbling.
109. Which of the following signs and symptoms would the nurse assess
in a client diagnosed with acute prostatitis?
A. Pain and swelling of the testicles
B. Inguinal and scrotal pain and edema
C. Hematuria and firm, enlarged prostate
D. Sudden onset of fever and dysuria.
110. If a client diagnosed with pelvic relaxation disorder was unable to
have surgical repair, what would be the best medical treatment of
choice?
A. Radiation therapy
B. Itrauterine device (IUD)
C. Bed rest
D. Pessary





DISORDERS OF PERCEPTION AND COORDINATION
111. An unconscious client with multiple injuries arrives in the
emergency department. Which nursing intervention receives the
highest priority?
A. Establishing an airway.
B. Replacing blood loss.
C. Stopping bleeding from open wounds.
D. Checking for a neck fracture.
112. What should the nurse do first when a client with a head injury
begins to have clear drainage from his nose?
A. Compress the nares.
B. Tilt the head back.
C. Give the client tissues to collect the fluid.
D. Administer an antihistamine for postnasal drip.
113. Which of the following nursing interventions is appropriate for a
client with an increased intracranial pressure of 20mmHg?
A. Give the client a warming blanket.
B. Administer low-dose barbiturates.
C. Encourage the client to hyperventilate.
D. Restrict fluids.
29

114. A client is being admitted with a spinal cord injury at C7. Which of
the following assessments take priority upon the clients arrival?
A. Reflexes, blood pressure, respirations
B. Blood pressure, temperature, respirations
C. Reflexes, bladder function, blood pressure, temperature
D. Reflexes, blood pressure, bladder function, respirations,
temperature
115. A male client with a head injury regains consciousness after several
days. Which of the following statements is most appropriate as the
client awakens?
A. Ill get your family.
B. Can you tell me your name and where you live?
C. Ill bet youre a little confused right now.
D. You are in the hospital. You were in an accident and
unconscious.
116. Which of the following will the nurse observe in the client in the
ictal phase of a generalized tonic-clonic seizure?
A. Jerking in one extremity that spreads gradually to adjacent
areas.
B. Vacant staring and abruptly ceasing all activity.
C. Facial grimaces, patting motions, and lip smacking.
D. Loss of consciousness, body stiffening, and violent muscle
contractions.
117. It is the night before patient Carla is to have a computed
tomography (CT) scan of the head without contrast. Which
statements of the nurse EJ would be most appropriate?
A. You must shampoo your hair tonight to remove all oil and
dirt.
B. You may drink fluids until midnight, but after that drink
nothing until the scan is completed.
C. You will have some hair shaved to attach the small
electrode to your scalp.
D. You will need to hold your head very still during the
examination.
118. What nursing assessments should be documented at the beginning
of the ictal phase of a seizure?
A. Heart rate, respirations, pulse oximeter, and blood
pressure
B. Last dose of anticonvulsant and circumstances at the time
C. Type of visual, auditory, and olfactory aura the client has
experienced.
D. Movement of the head and eyes and muscle rigidity.
119. Which clinical manifestation does nurse Diony expect in client Mana
in the postictal phase of generalized tonic-clonic seizure?
A. Drowsiness
B. Inability to move
C. Paresthesia
D. Hypotension
120. During the first 24 hours after thrombolytic treatment for ischemic
stroke, the primary goal is to control the clients:
A. Pulse
B. Respirations
C. Blood pressure
D. Temperature


RESEARCH
121. In the study regarding contraceptive counseling, the researcher
used a design which consists of two experimental groups and 2
control groups. One experimental group and one controlled group
would be administered the pretest and the other groups would not.
This is called:
A. Basic experimental design
B. Factorial design
C. Solomon four group design
D. Repeated measure design
122. An experiment regarding the effectiveness of auditory stimulation
in the development of premature infants is conducted in the
hospital of the sacred heart. The types of simulation are given at
different duration of exposures (15 minutes, 30 minutes, and 45
minutes). Multiple hypotheses are being formulated and tested.
What type of design was used?
A. Repeat measures design
B. Factorial design
C. Solomon four group design
D. After-only design
123. With quantitative instruments. It has become a customary
procedure for the developers to estimate the validity and the
reliability of their tools before making them available for general
use. Such an evaluation of an instrument is often referred as:
A. Stepwise replication
31

B. Psychometric assessment
C. Audit trail
D. Inquiry auditing
124. Diyosa conducted a research regarding the incidence and causes of
break ups among lovers in college who are in a relationship for less
than a year. He utilized structured data collection methods like
interview, observations, etc. he is utilizing what type of
triangulation?
A. Data
B. Theory
C. Method
D. Analysis
125. A research group wanted to conduct a study regarding the
influence of the 20-year Marcos reign on the politics of today, what
type of research fits this study?
A. Phenomenology
B. Historical research
C. Qualitative content analysis
D. Delphi survey
126. Which of the following data is a ratio measurement?
A. Quantity of sleep
B. Sleep loss cycle
C. Gender
D. Grades in school
127. The name and address of the research participants have been
expunged from the research report; this is in coordination with
what ethical aspect?
A. Confidentiality
B. Anonymity
C. Respect for human dignity
D. Justice
128. Anonymity is under what ethical principle according to the Belmont
report:
A. The Principle of Beneficence
B. Respect for Human Dignity
C. The principle of Justice
D. Code of ethics
129. The research participants have the following right when
participating in a study.
A. They have access to research personnel only during
scheduled interviews to clarify information.
B. Debriefing is not necessary at all as long as rapport is
established beforehand.
C. All agreements made between the researcher and the
participants are honored.
D. There is prejudicial treatment to individuals who decline to
participate from the study after agreeing to participate.
130. When conducting a study, a nurse researcher knows that fully
informed consent involves the disclosure of the following pieces of
information to participants except:
A. Participants status
B. Study purpose
C. Potential risk or costs
D. Contact information of the researcher is optional.









ANSWERS AND RATIONALES
1. A. INCORRECT. She volunteered as nurse to give care to injured
soldiers in military hospital.
B. INCORRECT. She is an abolitionist and worked as a nurse and
counselor for the Freedmens Relief Association.
D. INCORRECT. She was the Unions Superintendent of female
nurses during the Civil War.

C. CORRECT. She was known as The Moses of Her People. She
worked with the Underground Railroad. She nursed the sick and
suffering of her own race.

2. A. INCORRECT. Fabiola is a wealthy matron of the Roman Empire.
C. INCORRECT. Tetonic Knights is a member of The Crusades.
D. INCORRECT. They were also a member of The Crusades who
dedicated themselves to the care of people with leprosy, syphilis
and chronic skin conditions.

33

B. CORRECT. They saw the formation of several orders of knights.
These brothers provided nursing care to their comrades. They also
built hospitals, the organization and management of which set a
standard for the administration of hospitals.

3. B. INCORRECT. Nursing leader active in the protest movement for
womens rights.
C. INCORRECT. Mahoney was the first African-American trained
nursed.
D. INCORRECT. Founder of planned parenthood who was
imprisoned for opening the first birth control information clinic in
Baltimore.

C. CORRECT. She was considered the founder of public health
nursing. Together with Mary Brewer, they were the first to offer
trained nursing services to the slums of New York. She founded
the Henry Street Settlement school nursing established as an
adjunct to visiting nursing

4. A. INCORRECT. Is an example of illness prevention
B. INCORRECT. Is an aesthetic or not needed for health for health
promotion or disease prevention.
D. INCORRECT. Focuses on disease detection

C. CORRECT. Nurses promote wellness in clients who are both
healthy and ill. Health promotion focuses on maintaining normal
status without consideration of diseases. This may involve
individual and community activities to enhance healthy lifestyles
and preventing accidents in home and workplace.

5. A. INCORRECT. Restoration of health from early detection to
recovery period.
B. INCORRECT. The goal of illness prevention is to maintain optimal
health by disease prevention.
C. INCORRECT. This area involves comforting and caring for people
of all ages who are dying.

D. CORRECT. The quality or state of being in good health. This
includes self-responsibility; daily decision making in the areas of
nutrition; stress management, physical fitness; preventive health
care, emotional health and most importantly: the whole being of
the individual.

6. A. INCORRECT. Communication is a basic role wherein the nurse
identifies client problems and communicates it to other members
of the team.
B. INCORRECT. Change agent is another role of a nurse wherein she
assist client in making modifications in their behaviors.
D. INCORRECT. As a nurse manager, she delegates activities at the
same time supervises and evaluates the performance of other
nurses and ancillary workers.

C. CORRECT. Nurse administrator is a type of an expanded career
role. Before a nurse becomes an administrator, she/he is to be
educationally prepared for the position. The nurse administrator
functions include budgeting, staffing, and planning programs.

7. B. INCORRECT. An example of in-service education to upgrade the
knowledge or skills of employees
C. INCORRECT. In-service education to inform nurses about new
piece of equipment
D. INCORRECT. A type of mandatory in-service program
C. CORRECT. Continuing education refers to formalized experience
designed to enlarge the knowledge or skills of practitioners.
Constant updating and growth are essential to keep abreast of
scientific and technological change and changes within the nursing
profession. Continuing education is the responsibility of each
practicing nurse.

8. A. INCORRECT. Inappropriate way to communicate with the client
B. INCORRECT. Ineffective way to communicate with the client
D. INCORRECT. Inappropriate because it violates privacy and does
not ensure correct translation

C. CORRECT. Arranging for an interpreter would be the best
practice when communicating with a client who speaks a different
language. Many agencies have a qualified interpreter who
understands the healthcare system and can reliably provide
assistance.

9. B. INCORRECT. Resolution is the actual phase where old needs and
goals are put aside and new ones are adopted.
35

C. INCORRECT. Orientation, the phase where client seeks help and
nurse assists to understand the problem.
D. INCORRECT. Identification assumes the client for dependence,
interdependence and in relation to the nurse independence

A. CORRECT. Exploitation enables the client to derive full value to
what the nurse offers. The client uses available services based on
self-interest and needs. Power shifts from the nurse to the client.

10. B. INCORRECT. Time and experience are necessary for developing
proficiency in the profession or career.
B. INCORRECT. Research and theory development do not have
performance as their primary focus wherein nursing provides
quality service to humans.
D. INCORRECT. Team and group practice can be a part of the career
in humanities, computer science, or rocket science.

A. CORRECT. Practice disciplines are field of study in which the
central focus is performance of a professional role. Nurses and
nursing practice are often subordinate to powerful institutional
forces and traditions, introduction of any framework that
encourages nurses to reflect on, question and think about what
they do provide an invaluable service.

11. A. INCORRECT. This only describes persons/clients as a
metaparadigm.
B. INCORRECT. It only defines environment as a metaparadigm.
C. INCORRECT. The primary focus of nursing care is health.

D. CORRECT. Person/client, environment, health and nursing are
relevant when providing care for any client whether in the
hospital, at home, in the community, or in elementary school
systems. These elements can be used to understand diseases,
conduct and apply research, develop nursing theories, as well as
implement the nursing process.

12. A. INCORRECT. Deals with disputes between an individual and the
society as a whole
C. INCORRECT. Right is a privilege or fundamental power to which
an individual is entitled
D. INCORRECT. Responsibility is an obligation association with a
right

B. CORRECT. Civil action is a type of legal action. A nurses civil
action may include tort and the nurse contracts, and if found
guilty will have to pay a sum of money.

13. A. INCORRECT. Is the action of a lawsuit
B. INCORRECT. Delegation is the transfer of responsibility for the
performance of an activity from one person to another.
C. INCORRECT. Causation is an element of nursing malpractice.

D. CORRECT. Credentialing is a process of determining and
maintaining competence in nursing practice. It is also one way in
which the nursing profession maintains standards of practice and
accountability for educational preparation of its members.

14. A. INCORRECT. If this client is alert and competent, she can provide
consent.
B. INCORRECT. Some cases, minors are allowed to give consent for
procedures such as blood donations, treatment for substance
abuse, treatment for mental health, treatment for STDs or
pregnancy.
C. INCORRECT. The client is alert and competent.

D. CORRECT. There are three exceptions of people who cannot
provide consent. These are minors, unconscious/injured persons,
and those who are mentally ill. The mentally ill are not allowed to
give consent if judges by professionals to be incompetent.

15. A. INCORRECT. It is an example of a felony
B. INCORRECT. It is an example of malpractice
D. INCORRECT. It is an example of an intentional tort.

C. CORRECT. A misdemeanor is an offense of a less serious nature
and is usually punishable by a fine or short-term jail sentence, or
both. Slapping a clients face could be charged with a
misdemeanor.

16. A. INCORRECT. It uses knowledge from other subjects and fields.
B. INCORRECT. It uses creativity.
D. INCORRECT. It uses critical thinking in making decisions.
37


C. CORRECT. This uses deductive reasoning by contrast. If a nurse
thinks that all children love peanut butter sandwiches, she would
then think that if the client is a child, the child will love peanut
butter sandwiches. This example is considered invalid because
nurses use critical thinking to analyze situations and establish
valid premises.

17. B. INCORRECT. Is an example of a problem-solving approach
C. INCORRECT. It is a step in decision-making.
D. INCORRECT. It is a step in decision-making.

A. CORRECT. The mind of a critical thinker is filled with
questions. The curious nurse may value tradition but is not
afraid to examine it. The nurse may apply this to the issues of
moving responsibility for a procedure among the members of
the health care team.
18. B. INCORRECT. The nurse did not inferred and concluded her action
C. INCORRECT. It would have been judgment or opinion if the nurse
did not consider the lifespan consideration of her client
D. INCORRECT. It would have been judgment or opinion if the nurse
did not consider the lifespan consideration of her client

A. CORRECT. The nurse based it from a fact. Elders often have a
poor circulation. Warming the hands of the client for a few
moments may help the nurse in obtaining blood sample.

19. A. INCORRECT. Personal habits help determine a clients lifestyle
C. INCORRECT. Occupational history helps determine potential for
future disease and accident
D. INCORRECT. Communication style allows the nurse to determine
appropriate emotions and gestures.

B. CORRECT. Client record is an example of a data source. The
nurse can review such records before interviewing the client.
Client records may contain data regarding the clients occupation,
religion and marital status.

20. A. INCORRECT. Palpating first may cause rupture of abdominal
aneurysms
B. INCORRECT. Auscultation is performed before percussion and
palpation, it may increase peristalsis.
D. INCORRECT. Percussion if done first may provide falsely negative
result abdominal bowel sound.

C. CORRECT. When performing an abdominal assessment, the
specific order for assessment techniques is inspection,
auscultation, percussion and palpation.

21. A. INCORRECT. The client is positioned is prone.
B. INCORRECT. The client is positioned in supine
D. INCORRECT. The client is positioned in dorsal recumbent.

C. CORRECT. The client is positioned in lithotomy. PAP smear is a
screening test used for cervical and uterine cancer. Positioning a
client in lithotomy will allow the HCP to assess the female
genitals, rectum, and the female reproductive tract.

22. A. INCORRECT. The nurse is performing a voice test.
C. INCORRECT. After checking for presence of foreign bodies, the
client will be instructed not to move the head during the
examination to avoid damage to the canal and tympanic
membrane.
D. INCORRECT. The nurse is to perform a tuning fork test.

B. CORRECT. Before performing an otoscopic exam and inserting
the speculum, initially the auditory canal is to be checked for
foreign body then finally instructing the client to stay still.

23. B. INCORRECT. Flat and unelevated change
C. INCORRECT. Wheal is a reddened localized collection of edema
fluid and is irregular in shape.
D. INCORRECT. A type of vesicle with pus

A. CORRECT. Vesicles or bulla is a circumscribed round or oval thin
translucent mass filled with serous fluid or blood. It is a primary
skin lesion that appears initially in response to change in
internal/external environment of the skin.

24. A. INCORRECT. Facial hair is likely to become coarser, not finer.
C. INCORRECT. The sense of smell becomes less, rather than more
acute.
39

D. INCORRECT. The respiratory rate and rhythm is regular at rest.

B. CORRECT. Visual acuity often lessens with age. It decreases as
the lens of the eye ages and becomes more opaque and loses
elasticity.

25. A. INCORRECT. Recalling series of numbers test immediate recall.
B. INCORRECT. Subtracting backwards from 100 tests attention
span and calculation skills
D. INCORRECT. Recalling childhood events tests remote (long-term)
memory

C. CORRECT. Recent memory includes events of current day. Short
term memory is often less efficient. Long term memory is usually
unaltered.

26. A. INCORRECT. Lub-dub sounds are normal and represent the S1 (1
st

heart sound) and S2 (2
nd
heart sound).
B. INCORRECT. A pericardial friction rub is described as scratchy,
leathery sound
D. INCORRECT. A click is described as an abrupt high-pitched
snapping noise.

C. CORRECT. A heart murmur is an abnormal heart sound. It is best
described as gentle, blowing, swooshing sound.

27. A. INCORRECT. Recording the exact location allows determination
what lymph node is affected
B. INCORRECT. Nipples are noted it is displaced or retracted.
D. INCORRECT. Consistency determines whether the mass is hard or
soft.

C. CORRECT. Perfusion is not used in assessing the breast or
axillae. Perfusion is the blood supply on a peripheral vascular area
(e.g. extremities).

28. A. INCORRECT. Normal findings
B. INCORRECT. Normal findings
D. INCORRECT. Normal findings

C. CORRECT. In preadolescence, there is no pubic hair except for
the fine body hair. The hair assumes the texture and curl of the
adult but is not as thick and does not appear on the thighs.

29. B. INCORRECT. Presence of a femoral pulse would not provide
confirmation that arterial flow exists below that point.
C. INCORRECT. Taking a thigh BP requires locating the popliteal
pulse.
D. INCORRECT. Checking the distal pulse is done first before
requesting assistance from another nurse.

A. CORRECT. The purpose of finding the popliteal pulse is to
provide information about arterial circulation to the legs. If pedal
pulse which is more distant that the popliteal is present, then
adequate arterial circulation in the leg is present even though the
popliteal artery has not been located.

30. A. INCORRECT. Health examination entails complete head-to-toe
assessment of a body system or body part.
B. INCORRECT. Health assessment is done systematically and
requires fewest position changes for the client.
C. INCORRECT. Data obtained during a physical exam will
supplement, confirm, and refute data obtained during the nursing
history.

D. CORRECT. Initial assessment findings provide baseline data
about the clients functional abilities against which subsequent
assessment findings are compared.

31. A. INCORRECT.
B. INCORRECT.
C. INCORRECT.

D. CORRECT. The nurse is responsible for interpreting the
physicians written drug order, which frequently contains
abbreviations. The correct way of reading a physicians order is:
Administer/Give, the name of the drug, required dose, route of
medication, and frequency.

32. A. INCORRECT.
B. INCORRECT.
C. INCORRECT.
41


D. CORRECT. If any of these facts are missing, the order is
incomplete and should be questioned by the nurse. This prevents
overriding or conflicting orders and reduces the chance of
administration errors.

33. A. CORRECT
Solution:
30 mEq = x (mL)
20 mEq 15 mL
20x = (30 x 15)
20 20
x = 22.5 mL

34. A. CORRECT
Solution: First convert the desired dosage and the available dosage
to the same unit of measure (50mcg = 0.05mg). With the desired
dosage and the available dosage in the same unit of measure. The
problem can be set up by using the proportion:
Required dose x Desired Amount
Stock on hand Amount on hand

0.04 mg x 1mL
0.05 mg
= 0.8mL

35. B. CORRECT. To administer N70/30 insulin, 76 units SC by
injections from a 100 unit/mL multidose vial, the nurse should
withdraw 76 units using the 100 unit insulin syringe and 0.76mL
using the tuberculin syringe.

36. C. CORRECT.
Solution:
a. Check for the safe dose
Recommended ave. dose = childs dose
1 m
2
childs BSA
30mg = Childs dose
1 m
2
1.3 m
2

Childs dose = 30 mg (1.3)
= 39 mg
Using the available vial of the Methotrexate (25mg/mL) the nurse
will administer:
39 mg x 1mL = 1.56 or 1.6 mL
25 mg

37. D. CORRECT.
Solution:
Total volume (mL) x Drop Factor = Flow rate (gtts/min)
Total infusion time

100 mL = 10gtts
30 min mL
= 33 gtts/min


38. B. CORRECT.
Solution: Determine the mg for a person weighing 70 kg.
0.5mg x 70kg = 35 mg
Substitute:
Flow rate = Total volume (mL) x Drop Factor x amt of medication for
Total amount of medication body weight

= 100mL x 60 microdrops x 35 mg
2000 mg
= 105 microdrops per minute

39. C. CORRECT.
Solution:
Required dose x Desired Amount
Stock on hand Amount on hand

25 mEq = x
40 mEq 20mL
25(20) mL = x
40
500mL = x
40
12.5mL = x

40. C. CORRECT.
Solution: Required dose x Desired Amount
Stock on hand Amount on hand
43

= 500mL
250
= 2 mg of theophylline per mL
A solution of 1 mL of the prediluted theophylline 500mg in 250mL D5W
contains 2mg of theophylline per mL.
Convert: 2mg= _____g
2mg x 1000g = 2000g

Determine how many milliliters are equal to 750g:
750g = x
2000
0.375mL = x
Infusion rate:
60 min = x mL
1min 0.375mL
X= 60(0.375mL)/hr
X= 22.5 mL/hr

41. A. INCORRECT.
B. INCORRECT.
D. INCORRECT.

C. CORRECT. Action research involves utilizing a cyclical method of
planning, taking action, observing, educating and critical reflecting
before planning the next cycle.
42. B. INCORRECT. Social investigation is a process of systematically
learning and analyzing the various structures and forces in the
community.
C. INCORRECT. Mobilization refers to the activities done by the
community to solve problems confronting the community
affiliations which serve to build and strengthen the peoples self
confidence and collective spirit.
D. INCORRECT.

A. CORRECT. Groundwork involves motivating people. Motivation
is essential in change.

43. B. INCORRECT.
C. INCORRECT.
D. INCORRECT.

A. CORRECT. A reporting unit is defined as any DOH health care
facility that renders/delivers public-related services to targeted
beneficiaries. The lowest level of reporting unit is the Barangay
Health Station.

44. A. INCORRECT. Health care knowledge is focused on the application
of community health nursing process.
C. INCORRECT. Health care practices include case finding, direct
care, health counseling and guidance
D. INCORRECT.

B. CORRECT.

45. B. INCORRECT. Philippine Food Fortification Act of 2000
C. INCORRECT. Philippine Clean Air Act of 1999
D. INCORRECT. Fidel Salt Act of 1995

C. CORRECT. Newborn Screening Act

46. A. INCORRECT. Magna Carta of Public Health Workers
C. INCORRECT. National Health Insurance Act
D. INCORRECT. Senior Citizens Act

B. CORRECT. Local Government Code

47. A. INCORRECT.
C. INCORRECT.
D. INCORRECT.

B. CORRECT. Self-willed changes have more meaning and
permanence than imposed changes.

48. A. INCORRECT. A type of strategy adopted to achieve the goals and
objectives of family planning.
B. INCORRECT. A type of strategy adopted to achieve the goals and
objectives of family planning.
C. INCORRECT. A type of strategy adopted to achieve the goals and
objectives of family planning.

D. CORRECT.

45

49. A. INCORRECT.
C. INCORRECT. Integration is under entry phase.
D. INCORRECT.

B. CORRECT. Area selection and community study which includes
demographic and geographical profile are activities done under
preparatory phase.

50. A. INCORRECT.
B. INCORRECT.
D. INCORRECT.

C. CORRECT. After greeting and making the client feel at ease the
nurse should take the clinical history of the patient. These are
observed by the nurse during medical examination.

51. B. INCORRECT. Definition of community competence
C. INCORRECT. Definition of community competence
D. INCORRECT. Definition of participation

A. CORRECT.

52. A. INCORRECT.
C. INCORRECT.
D. INCORRECT.

B. CORRECT. Supervision function is directed towards lower level
health workers, since they require closer supervision than
professional health workers.

53. A. INCORRECT.
B. INCORRECT.
C. INCORRECT.

D. CORRECT. It is a cyclical method.

54. A. INCORRECT. It is a primary prevention since it prevents a
reservoir from harbouring a known vector of dengue.
B. INCORRECT. BCG prevents primary complex therefore, a primary
prevention.
C. INCORRECT. Tertiary prevention since it prevents complications
after an illness.

D. CORRECT.

55. A. INCORRECT. Maintaining strict isolation is not needed because
human immunodeficiency virus is spread by contact with
contaminated blood/body fluids, which can be avoided by following
standard precautions.
B. INCORRECT. A private room would not provide barrier
protection, which is needed for standard precautions.
D. INCORRECT. Wearing a gown is appropriate only when
anticipating splashing blood/body fluids

C. CORRECT. Standard precautions stipulate that a health care
worker wear gloves when contact with clients blood/body fluids
in anticipated, such as when providing oral care.

56. A. INCORRECT. Excessive intake of protein will not cause
autoimmunity.
C. INCORRECT. Excessive intake of vitamin A is not related to
decrease immunologic function.
D. INCORRECT. Decreased intake might cause immune dysfunction

B. CORRECT. A diet containing excessive fat seems to contribute to
autoimmunity-over reaction of the body against constituents of its
own tissues. Immune dysfunction has been linked to deficient
intake of proteins, vitamin A, and zinc.

57. A. INCORRECT.
B. INCORRECT.
D. INCORRECT.

C. CORRECT. Oral acyclovir may cause GI effect such as diarrhea,
nausea and vomiting.

58. A. INCORRECT.
B. INCORRECT.
C. INCORRECT.

D. CORRECT. Prophylactic isoniazid therapy must continue for 6-12
months at a daily dose of 300mg. Taking the drug for less than 6
47

months may not provide adequate protection against
tuberculosis.

59. A. INCORRECT.
B. INCORRECT.
C. INCORRECT.

D. CORRECT. Methotrexate is a folic acid antagonist. Leucoverin is
the drug given for toxicity to this drug. It is used to treat iron
deficiency anemia, create a synergistic effect, or increase the
number of circulating neutrophils.

60. B. INCORRECT. Mansonia bonnea is a type of vector but not the
principal vector in Filariasis.
C. INCORRECT. Anopheles flavivoris is a secondary vector.
D. INCORRECT. Mansonia uniformis is a vector but not considered
as a principal vector.
C. CORRECT. Aedes poecilus is the principal vector.

61. A. INCORRECT. The causative agent for mumps is mumps virus
(Paramyxoviridae)
B. INCORRECT. The causative agent for diphtheria is
corynebacterium diphtheria.
C. INCORRECT. Pertussis has a causative agent named Hemophilus
pretussis/ Bordetella pertussis

D. CORRECT. Varicella zoster virus is the causative agent of
chicken pox.

62. A. INCORRECT. Covering the nose and mouth when sneezing or
coughing protects others but cannot protect the client from getting
the flu.
B. INCORRECT. Antibiotics, which work against bacteria, are not
effective against influenza virus.
D. INCORRECT. Submitting to diagnostic procedures cannot prevent
the client from contracting influenza.

C. CORRECT. Vaccination is the best method of protecting against
influenza infection.

63. A. INCORRECT. Infection of the scalp
C. INCORRECT. Tinea cruris id for infection of the upper thigh and
inguinal creases
D. INCORRECT. Infection of the foot

B. CORRECT. Upper arm is found in the body. Tinea corporis
indicates infection in the body.

64. A. INCORRECT. Adult is the last stage living about 30 days
D. INCORRECT. Nymphs are the newly hatched lice and become
adult in 8-9 days.
C. INCORRECT. Newly laid eggs are small, translucent and difficult to
see.

B. CORRECT. The mother is finding empty egg shells. Eliminating
the nits will help in eradicating hair lice.

65. A. INCORRECT. Lochia on the 2
nd
day is a normal finding.
C. INCORRECT. The WBC count of a postpartum woman is not of
great value.
D. INCORRECT. Breast engorgement is a normal response in
postpartum and is not associated with endometritis.

B. CORRECT. Fever on the third/fourth day postpartum should
raise concerns about possible endometritis until proven. A woman
with endometritis normally presents with a temperature over
38C.

66. C. INCORRECT. Primary dysmenorrheal refers to the menstrual pain
without identified pathology.
B. INCORRECT. Mittleschmerz refers to the pelvic pain that occurs
midway between menstrual periods
D. INCORRECT. Amenorrhea is the cessation of menstruation for a
period of at least 3 cycles or 6 months in a woman who has
established a pattern of menstruation and can result from a variety
of causes.

A. CORRECT. Endometriosis is defined as the presence of tissue
outside the uterus that resembles the endometrium in both
structure and function. The response of this tissue to the
stimulation of estrogen and progesterone during the menstrual
cycle is identical to that of the endometrium.
49


67. A. INCORRECT.
B. INCORRECT.
D. INCORRECT.

C. CORRECT. Fluid volume during pregnancy peaks between 18-32
weeks gestation. During this period, it is essential to observe and
record maternal data that would indicate further signs of cardiac
decompression and CHF. Assessing the lung sounds may identify
early symptoms of diminished oxygen exchange and potential
CHF.

68. A. INCORRECT. IV fluids will be initiated after oxygen care is
rendered.
C. INCORRECT. The client will be positioned per physicians order.
D. INCORRECT. Although CS birth may be needed, there is no data
that is necessary at this time.
C. CORRECT. Administering oxygen will increase the amout of
oxygen for transport to the fetus, partially compensating for the
loss of circulating blood volume. This action is essential regardless
of the cause or amount of bleeding.

69. A. INCORRECT. Not every client with HIV has problems with self-
care.
C. INCORRECT. Nutritional deficit can be a concern but not priority.
D. INCORRECT. Activity is not a problem for some clients with HIV.

B. CORRECT. Clients with HIV often show some evidence of
immune dysfunction and may have increased vulnerability to
common infections. HIV infection impairs cellular and humoral
immune function; therefore individuals with HIV are vulnerable to
common bacterial infections. Infection is specifically related to
HIV and is a priority because it is more life-threatening.

70. A. INCORRECT. Systolic murmurs usually indicate the presence of a
PDA, which is a common complication of RDS.
B. INCORRECT. Respiratory rates above 60 are indicative of
tachypnea which is a sign of respiratory distress.
C. INCORRECT. Edema of the hands and feet occurs within the first
24 hours as a result of low protein concentration, a decrease in
colloidal osmotic pressure and transudation of fluid from the
vascular system to tissues.

D. CORRECT. Increase urination is a n early sign that the neonates
respiratory condition is improving. Lung fluid, which the condition
improves and the alveoli open. This extra fluid circulates to the
kidneys, which results in increased voiding.

71. A. INCORRECT. Pacifiers could damage the operative site.
B. INCORRECT. A baby in a prone position may rub her face on the
sheets and traumatize the operative site.
D. INCORRECT. The suture line should be gently cleaned gently to
prevent infection, which could interfere with healing and damage
cosmetic appearance of the repair. Dried blood collecting on the
suture line can widen the scar.

C. CORRECT. Soft restraints from the upper arm to the wrist
prevent the infant from touching his lip but allow him to hold a
favorite item such as blanket.

72. B. INCORRECT.
D. INCORRECT. Vitamin K is administered to the newborn infant to
prevent abnormal bleeding and it promotes liver formation of the
clotting factors.
C. INCORRECT.

A. CORRECT. Opthalmic erythromycin is a broad spectrum
antibiotic and is used prophylactically to prevent opthalmia
neonatorum. Erythromycin is effective against Neisseria
gonorrheae and Chlamydia trachomatis

73. B. INCORRECT. Amphotericin is used for fungal infections.
C. INCORRECT. Gancyclovir is used to treat cytomegalovirus.
D. INCORRECT. Amantadine is used to treat influenza A virus.

A. CORRECT. The nursing care of a child with RSMF will include the
administration of tetracycline. An alternative medication is
chloramphenicol, a fluoroquinolone.

74. B. INCORRECT. Water is used for cleaning because soap or baby
wipes may irritate the area and cause discomfort.
51

C. INCORRECT. Vaseline gauze should be reapplied; frequent diaper
changing prevents contamination of the site.
D. INCORRECT. The mother should call the physician if the baby has
not urinated for the next 24 hours because swelling or damage may
obstruct urine output.

A. CORRECT. The mother should be taught to observe for bleeding
and to assess the site hourly for 8-12 hours following the
circumcision.

75. A. INCORRECT. Treatment is started as soon as possible after birth.
B. INCORRECT. Children with clubbed foot can recur thus, requiring
long-term interval follow-up until they reach skeletal maturity to
provide outcome.
D. INCORRECT. Serial manipulation and casting are performed at
least weekly.

C. CORRECT. Clubfoot is a complex deformity of the ankle and foot
that includes forefoot adduction, midfoot supination, hindfoot
varus, and ankle equinus. The defect may be unilateral or
bilateral. If sufficient correction is not achieved in 3 to 6 months,
surgery usually is indicated.


76. A. INCORRECT. Slowing the infusion rate is done in dialysis.
C. INCORRECT. Slowing the infusion is not done in BT.
D. INCORRECT. Do not administer acetaminophen.

B. CORRECT. The symptoms suggest transfusion reaction. The
priority nursing intervention for a client with symptoms of an
acute hemolytic reaction is to stop the blood transfusion
immediately.

77. A. INCORRECT. Exposure to cold and subsequent constriction, not
exposure to warmth that causes dilation, may precipitate angina
pain.
C. INCORRECT. Rest typically relieves anginal pain, except that
anginal pain is caused by vasospasm.
D. INCORRECT. Light meals are recommended, heavy meals should
be avoided because increased oxygen is needed to digest food.

B. CORRECT. Any activity that increases myocardial oxygen
demands, such as smoking can lead to angina pain. Smoking also
can cause vasoconstriction that can precipitate angina attacks.

78. A. INCORRECT. Assessing her blood pressure more frequently would
be appropriate if the vital signs are unstable.
B. INCORRECT. False or empty reassurance does not meet the
clients need
D. INCORRECT. Explaining that another episode is unlikely or telling
her not to worry would block further communication.

C. CORRECT. Because the clients vital signs are stable, the client is
most likely experiencing anxiety related to the acute event. The
nurse should encourage the client to express her feelings about
the MI to help reduce the clients anxiety level.

79. A. INCORRECT. Weight gain may be a result of fluid retention from
possible non-compliance with medications.
C. INCORRECT. Significant improvement in activity level usually is
not possible for clients with CHF.
D. INCORRECT. Medication teaching classes is but one aspect of the
clients discharge plan. Although attendance at the classes may help
with compliance, it does not ensure the clients compliance.

B. CORRECT. Less frequent hospital admissions indicate that the
client is experiencing better heart function and therefore must be
complying with the discharge plan.

80. A. INCORRECT. A high fiber is encouraged to help decrease the
cholesterol level.
B. INCORRECT. A client should avoid eating 2 hours before exercise,
because digestion increases blood supply to the GI system and
decreases supply to the heart muscles.
C. INCORRECT. The client must quit smoking, not just decrease the
amount.

D. CORRECT. The client should participate in a regular exercise
program, such as walking, swimming, or low-impact aerobics for
at least 20-30 minutes a day plus warm up and a cool down time.

81. A. INCORRECT. Pale skin may co-exist with angina
53

B. INCORRECT. Pale skin may coexist with MI
D. INCORRECT. Weakness, fatigue, fever, diaphoresis, athralgia, and
petechiae typically are evidence of endocarditis.

C. CORRECT. Frothy blood-tinged sputum appearing in conjunction
with dyspnea and clammy, pale skin indicates pulmonary edema
with interstitial fluid overload in the lungs because of left
ventricular failure.

82. A. INCORRECT. Valuable time is wasted by assessing blood pressure.
C. INCORRECT. Time is wasted in auscultating heart sounds.
D. INCORRECT. Defibrillation is used after initial resuscitation efforts
have been initiated.

B. CORRECT. If a client experiences cardiac arrest, the first action
is to establish a patent airway and then administer artificial
ventilation and oxygen. Resuscitation measures need to be
started immediately because of the life-threatening nature of a
cardiac arrest.

83. B. INCORRECT. Second-degree AV block is a conduction defect
evidenced by a slow-rate, progressively lengthening or fixed PR
interval, and normal P wave and QRS complex.
C. INCORRECT. VTach is evidenced by a rate of 100 to 250bpm, no
PR interval, wide bizaare QRS complex and abnormal conduction
through ventricular tissue.
D. INCORRECT. Sinus bradycardia refers to a heart rate less than
60bpm with all other ECG waveforms within normal parameters.

A. CORRECT. Irregular radial pulses in conjunction with up to a 10-
beat difference between the apical and radial pulse rates indicate
atrial fibrillation.

84. A. INCORRECT. A structured exercise program involving daily low-
impact aerobic exercises also would be included but of less priority.
B. INCORRECT. Dietary instructions should address an intake of low
sodium, high potassium foods, especially if the client is receiving
diuretic therapy.
D. INCORRECT. Discussion of possible surgery is inappropriate for a
client with chronic CHF.

C. CORRECT. For the client with chronic CHF, teaching topics must
include the signs and symptoms of pulmonary edema. This
condition is a medical emergency situation requiring prompt
evaluation and treatment. Otherwise, it could progress to death.

85. A. INCORRECT. Reassuring the client is inappropriate.
C. INCORRECT. Elevating the leg would impede arterial circulation
to the foot.
D. INCORRECT. Ambulation and calf muscle exercise cause further
tissue ischemia.

B. CORRECT. The nurses assessment findings indicate an acute
occlusion of the arterial circulation, possible by an embolus or
thrombus after angiography. This necessitates immediate
intervention to restore circulation to the leg.

86. B. INCORRECT. Weakness, fatigue and petechiae of the anterior
trunk and conjunctiva indicate endocarditis.
C. INCORRECT. Crushing chest pain radiating down the left arm
accompanied by nausea and vomiting may be associated with a
myocardial infection.
D. INCORRECT. Hepatomegaly, jugular vein distention and
dependent peripheral edema are manifestations of right-sided
heart failure.

A. CORRECT. Sharp sudden pain over the precordium that radiates
to the left scapular region is a characteristic clinical manifestation
of pericarditis and inflammation of the pericardium. The pain may
be aggravated by breathing or movement.

87. A. INCORRECT. Reteaching the client with medication may or may
not be effective.
C. INCORRECT. Teaching may help with compliance.
D. INCORRECT. Arranging for outpatient follow up may help with
compliance but may cause client to feel loss of control over
condition and its management.

B. CORRECT. To ensure compliance, the nurse needs more
information to determine if there is specific reason why the client
is not complying with the medication regimen. However, financial
55

constraints or accessibility to the pharmacy needs to be identified
or addressed.

88. B. INCORRECT. Nicotine can cause vasoconstriction.
C. INCORRECT. These are areas assessed for any client.
D. INCORRECT. These are areas assessed for any client.

A. CORRECT. Intermittent claudication, calf pain associated with
exercise, is symptomatic of ischemia caused by arterial occlusive
disorders in the legs. Leg color, temperature, pulses and pain
could be used to ascertain clinical symptoms of arterial problems.

89. A. INCORRECT. Position does nothing to improve activity tolerance.
B. INCORRECT. Improved peripheral blood flow and tissue perfusion
may lead to decrease pain.
C. INCORRECT. Disuse syndrome is not typically associated with
chronic arterial occlusive disease.

D. CORRECT. The client with chronic arterial occlusive disease of
the lower extremities experiences an alteration in the tissue
perfusion to this area. As a result, placing the legs in a dependent
position in relation to the heart helps to improve peripheral blood
flow.

90. A. INCORRECT. Hamburger contains high cholesterol.
B. INCORRECT. Liver is high in cholesterol.
D. INCORRECT. Milkshakes and fried foods tends to have high
cholesterol content.

C. CORRECT. Pasta, tomato sauce, salad, and coffee would be the
best selection for the client following a low-cholesterol diet.

91. A. INCORRECT. Bulk laxatives and increased fluid intake help
prevent exacerbation of diverticulosis.
B. INCORRECT. A high fiber diet would further irritate the bowel. A
rectal tube is not required.
D. INCORRECT. After pain subsides, client should resume eating a
low residue diet.

C. CORRECT. During an episode of acute diverticulitis, the bowel
must be totally at rest, and the client therefore must be kept on
NPO. Nasogastric suctioning helps decompress the bowel.

92. A. INCORRECT. Mid-epigastric and tenderness suggest peptic ulcer
diseases.
B. INCORRECT. Abdominal distention and fever are associated with
peritonitis.
C. INCORRECT. Abdominal cramping and vomiting are commonly
seen with gastroenteritis.

D. CORRECT. Common clinical manifestations of esophagitis
include heartburn, acid regurgitation, belching, dyphagia and
esophageal pain radiating to arms, neck and jaw.
93. A. INCORRECT. The head of the bed is the one that should be
elevated.
B. INCORRECT. The client should sit up or remain upright after
eating.
C. INCORRECT. Diarrhea is not associated with GERD.

D. CORRECT. For the client with GERD, anything that can increase
gastric acid production should be avoided, including caffeine,
tobacco, peppermint, chocolate, onions, and fatty or fried foods.

94. A. INCORRECT. Diverticulitis commonly complains of a LLQ pain.
C. INCORRECT. Gastroenteritis is often manifested by abdominal
cramping, nausea, vomiting and diarrhea.
D. INCORRECT. A client with irritable bowel syndrome typically
complains of bowel habits and pain usually located in the LLQ that
is relieved by passing flatus or stool.

B. CORRECT. Severe RLQ pain, nausea, vomiting, and low grade
fever are common clinical manifestations of appendicitis.

95. A. INCORRECT. Corticosteroids do not cause photosensitivity.
B. INCORRECT. Moon face and buffalo hump are signs of overdose
but should not stop taking the medication.
C. INCORRECT. This medication should be taken with food to
minimize risk for gastric upset.

D. CORRECT. Corticosteroid suppresses adrenal gland function.
When these medications are to be discontinued, the dose must be
57

tapered so that the adrenal gland will resume adequate
functioning. Otherwise adrenal insufficiency may occur.

96. B. INCORRECT. Watery diarrhea indicates overdose.
C. INCORRECT. Daily deterioration in the clients handwriting
indicates increase in the ammonia level and worsening of hepatic
encephalopathy
D. INCORRECT. A frothy, foul-smelling stool indicates steatorrhea,
caused by impaired fat digestion.

A. CORRECT. Lactulose reduces serum ammonia levels by inducing
catharsis, subsequent decreasing colonic pH and inhibiting fecal
flora from producing ammonia from urea. Ammonia is removed
with the stool. Two or three soft stools daily indicate effectiveness
of the drug.

97. B. INCORRECT. The tube is indicated for short-term use and in not
advised to use at home.
C. INCORRECT. The client would not be helping to insert the tube.
D. INCORRECT. A clients anxiety should be decreased not
maintained.

A. CORRECT. An esophageal tamponade tube would be inserted in
critical situations. Typically, the client is fearful and highly anxious.
The nurse therefore explains about the placement to help obtain
the clients cooperation and reduces fear.

98. A. INCORRECT. Vasopressin is appropriate if the client is diagnosed
with bleeding esophageal varices.
B. INCORRECT. Paracentesis and diuretics would be appropriate if
the client is diagnosed with portal HPN and ascites.
D. INCORRECT. A low-fat diet and increased fluid intake would
further aggravate the pancreatitis.

C. CORRECT. With acute pancreatitis, the client is kept NPO to
inhibit pancreatic stimulation and secretion of pancreatic
enzymes. NGT with low intermittent suction is used to relieve
nausea and vomiting, decrease painful abdominal distention and
remove hydrochloric acid.

99. B. INCORRECT. Ecchymosis, petechiae and coffee-ground emesis are
clinical manifestations of esophageal bleeding.
C. INCORRECT. Jaundice, dark urine, steatorrhea are clinical
manifestations of icteric phase of hepatitis.
D. INCORRECT. Pain of cholecystitis is typically located in the RUQ,
not in RLQ. Diarrhea and dehydration are not common in acute
cholecystitis.

A. CORRECT. Acute cholecystitis is an acute inflammation of the
gall blasser commonly manifested by anorexia, nausea and
vomiting, biliary colic, tenderness and rigidity in the RUQ, fever,
fat intolerance and signs and symptoms of jaundice.

100. B. INCORRECT. Applying pressure when giving intramuscular
injections if client have bleeding problems.
C. INCORRECT. Decreasing the clients dietary protein intake is
appropriate if clients ammonia levels are increased.
D. INCORRECT. Vitamin K administration is appropriate if there are
bleeding problems.

A. CORRECT. The client with pruritus experiences itching, which
may lead to skin breakdown and possibly infection from
scratching.

101. A. INCORRECT. Signs and symptoms of UTI include flank pain, fever,
chills, and dysuria.
B. INCORRECT. Clinical manifestations of renal calculi are acute,
sharp, and intermittent pain along with nausea and vomiting.
C. INCORRECT. Altered urine output, hypertension and tachypnea
are manifestations of acute renal failure.

D. CORRECT. Clotting or bed wetting, dribbling, urgency, hesitancy
or an inability to get to the bathroom are clinical manifestations of
incontinence.

102. A. INCORRECT. Incorrect formulation
C. INCORRECT. Fluid intake should be divided over a 24-hour period
D. INCORRECT. Drinking 500mL fluid four times a day exceeds the
recommended water intake.

B. CORRECT. Insensible losses (500mL) plus urine output
determine intake in renal failure.
59


103. B. INCORRECT. A bedridden, elderly client with an indwelling foley
catheter would be prone to UTI.
C. INCORRECT. 18-month old toddler with vesicoureteral reflux is
more prone to UTI
D. INCORRECT. A 55 year old woman would be more prone to UTI
than a young adult man.

A. CORRECT. Kidney infections are caused by immobility, reflux,
stasis and debilitation. Women are more prone to UTI than men.

104. A. INCORRECT. Urinary stress incontinence and abdominal pain are
not signs and symptoms of acute pyelonephritis.
C. INCORRECT. Burning on urination is does not manifest acute
pyelonephritis
D. INCORRECT. Acute sharp intermittent pain and anuria are not a
signs and symptoms that support the diagnosis

B. CORRECT. Common clinical manifestations of acute
pyelonephritis include flank pain, fever, chills, dysuria,
costovertebral angle ternderness, frequency and urgency, malaise
and possibly bloody or cloudy urine.

105. A. INCORRECT. The client cannot leave the room
B. INCORRECT. Visitors are not allowed because he is risk for
infection and must be kept in isolation
D. INCORRECT. Discussing activities that the client can do to combat
isolation is appropriate but the nurse must allow the client to
verbalize his feelings

C. CORRECT. An immunosuppressed post-tranplantation client is
placed on mask isolation in a single room and protected from
anyone with an infection. Because the client expressed
frustration, the nurse should allow the client to verbalize his
feelings.

106. B. INCORRECT. Keeping a diary of menstrual cycle have no effect in
muscle tone
C. INCORRECT. Administration of vaginal cream is will not help
D. INCORRECT. Frequent intercourse will not have an effect

A. CORRECT. A pelvic relaxation disorder is a structural disorders
resulting in a weakening of the support tissues. The goal is to
strengthen the surrounding muscles. Kegels exercise will help
strengthen perineal area.

107. A. INCORRECT. Discussing ways to prevent a sickle cell crisis is
appropriate for client with priaprism.
B. INCORRECT. Applying ice pack will help if the client has infection
D. INCORRECT. Elevating the scrotum is appropriate if with scrotal
edema.

D. CORRECT. In phimosis, the foreskin is constricted and cannot be
retracted. Poor hygiene of the uncircumcised penis is most
appropriate to prevent recurrence.

108. B. INCORRECT. Rectal exam is important for rectal or prostate
cancer.
C. INCORRECT. Aspirin is expected if client has CAD
D. INCORRECT. Frequency and dribbling are signs and symptoms of
BPH.

A. CORRECT. Impotence may result from psychogenic and organic
causes. Endocrine conditions such as DM, pituitary tumors and
hypogonadism are possible organic cause of impotence.

109. A. INCORRECT. Testicular pain and swelling are associated with
orchitis
B. INCORRECT. Inguinal and scrotal swelling are associated with
epididymitis
C. INCORRECT. Hematuria suggests enlarged prostate

D. CORRECT. Signs and symptoms of acute prostatitis include
sudden onset of fever and chills, dysuria and urgency.

110. A. INCORRECT. Not an appropriate therapy for structural disorders
B. INCORRECT. Not an appropriate therapy for structural disorders
D. INCORRECT. Not an appropriate therapy for structural disorders

C. CORRECT. Pelvic relaxation disorders are structural disorders
resulting in weakening of support tissues. If they cannot be
corrected surgically, the medical treatment of choice is pessary.
61


111. B. INCORRECT. Replacing blood loss will be completed after airway
C. INCORRECT. Stopping bleeding from open wounds will be
completed after ventilation and airways are established
D. INCORRECT. Checking for a neck fracture will be done later

A. CORRECT. The highest priority for a client with multiple injuries
is to establish an airway for effective ventilation and oxygenation.
Unless the client has a patent airway, other care measures would
be futile.

112. A. INCORRECT. Compressing the nares will obstruct the drainage
flow.
B. INCORRECT. It is appropriate to tilt the head back which would
allow draining of fluid on the throat and not be collected for
sample.
D. INCORRECT. It is inappropriate because the drainage may not be
postnasal drip.

C. CORRECT. The clear drainage must be analyzed to determine
whether it is a nasal drainage or cerebrospinal fluid (CSF). The
nurse should give the client tissues because it is important to
know how much leakage of CSF is occurring.

113. A. INCORRECT. A cooling blanket is used to control the elevation of
temperature.
B. INCORRECT. High doses of barbiturates may be used to reduce
increased cellular metabolic demands
D. INCORRECT. Fluid volume and inotropic drugs are used to
maintain cerebral perfusion by supporting the cardiac output

C. CORRECT. Normal ICP is 15mmHg or less for 15 to 30 seconds or
longer. Hyperventilation causes vasoconstriction, which reduces
CSF and blood volume, two important factors for reducing a
sustained ICP of 20mmHg.

114. A. INCORRECT.
C. INCORRECT.
D. INCORRECT.

B. CORRECT. The nurse should assess the client for spinal shock,
which is the immediate response to spinal cord transaction.
Hypotension occurs and the body loses core temperature to
environmental temperature. The nurse must treat the client
immediately to manage hypotension and hypothermia. The nurse
should also ensure that there is an adequate airway and
respirations. Once the client is stable, the nurse will conduct a
complete neurologic check. If the client is already conscious the
nurse then will check for reflexes.

115. A. INCORRECT. Offering to get his family is important after re-
orientation
B. INCORRECT. Asking the client questions is important to know if
he is oriented
C. INCORRECT. Making comments is not helpful and may cause
anxiety

D. CORRECT. It is important to first explain where a client is to
orient him to time, person, and place. Asking questions to
determine orientation is done after letting the client know where
he is and what happened to him.

116. A. INCORRECT. Partial seizure starts in one region of the cortex
B. INCORRECT. Absence seizure usually involves blank stares
C. INCORRECT. Complex partial seizure involves facial grimacing
with patting and smacking

D. CORRECT. A generalized tonic-clonic seizure involves both a
tonic phase and a clonic phase. The tonic phase consists of a loss
of consciousness, dilated pupils, and muscular stiffening. Clonic
involves repetitive movements.

117. A. INCORRECT. Shampooing the hair is not required
B. INCORRECT. Foods and fluids are withheld because contrast
medium causes nausea
D. INCORRECT. Electrodes are not used so having the head shaved is
unnecessary

D. CORRECT. The client will be asked to hold the head very still
during the examination which lasts about 30-60 minutes.

63

118. A. INCORRECT. Assessing clients pulse is impossible because of
muscle contractions
B. INCORRECT. The last dose of anticonvulsant can be evaluated
later
C. INCORRECT. Aura is assessed in the preictal phase of the seizure

D. CORRECT. During the seizure, the nurse should note movement
of the clients head and eyes and muscle rigidity, especially when
the seizure first begins, to obtain clues about the location of the
trigger focus of the brain.

119. B. INCORRECT. Inability to move is not expected
C. INCORRECT. A change in sensation would not be expected
because this would indicate a complication such as injury to the
peripheral nerve pathway
D. INCORRECT. Hypotension is not typically a problem after a
seizure.

A. CORRECT. The nurse should expect a client in the postictal
phase to experience drowsiness to somnolence because
exhaustion results from abnormal spontaneous neuron firing and
tonic-clonic motor response.

120. A. INCORRECT. Is important but not a priority
B. INCORRECT. Is important but not a priority
D. INCORRECT. Is important but not a priority

C. CORRECT. Control of blood pressure is critical during the first 24
hours after treatment because an intracerebral hemorrhage is a
major adverse effect of thrombolytic therapy.

121. A. INCORRECT. Basic experimental designs are the after-only design
and the before-after design
B. INCORRECT. Factorial designs permits testing of multiple
hypotheses in a single experiment
D. INCORRECT. Repeated measures design involves exposure of the
same subjects

C. CORRECT. This is done to allow the effect of the pretest
measure and intervention to be segregated.

122. A. INCORRECT. Repeated measures design involves exposure of the
same subjects
C. INCORRECT. Solomon four group designs consist of two
experimental groups and two control groups
D. INCORRECT. One experiment group and one control group would
not be administered pretest and the other groups not.

B. CORRECT. It promotes testing of multiple hypotheses in a single
experiment.

123. A. INCORRECT. Stepwise replication involves having a research
group that can be divided into two teams
B. INCORRECT. Audit trail is a systemic collection of materials and
documentation that will allow an independent auditor to come to
conclusions about.
D. INCORRECT. Inquiry audit involves scrutiny

C. CORRECT. Psychometric assessment estimates the validity and
reliability of tools

124. A. INCORRECT. Data triangulation involves multiple data sources
B. INCORRECT. Uses competing theories or hypothesis of the
analysis
D. INCORRECT. The use of 2 or more analytic techniques to analyze
the same set of data

C. CORRECT. Method triangulation involves the use of multiple
methods in collecting data about the same phenomenon.

125. A. INCORRECT. Phenomenology is the study of life experiences of
people
B. INCORRECT. Qualitative analyzes the content of the narrative to
determine themes and patterns
D. INCORRECT. Delphi survey involves panel of experts who are
asked to complete a series of questionnaires

B. CORRECT. Historical research is the systematic collection and
critical evaluation of data relating to past occurrence.

126. A. INCORRECT. Grades in school are ordinal and permits sorting of
objects according to standing
65

B. INCORRECT. Sleep loss scale is interval which specifies both
ranking of objects
D. INCORRECT. Gender is nominal. Nominal is the lowest level of
measurement which involves assignment of numbers to classify
characteristics into categories.

A. CORRECT. Ratio measurement is the highest level, ratio scales
are distinguished from interval scales by virtue of having a
rational, meaningful zero.

127. A. INCORRECT. A promise of confidentiality to the participants is a
pledge that any information that the participant provides will not
be publicly reported.
B. INCORRECT. Respect for human dignity includes the right to self
determination and the right to full disclosure.
D. INCORRECT. Justice is an ethical principle which encompasses the
principle of anonymity and confidentiality under the right to privacy

B. CORRECT. Anonymity occurs when even the researches cannot
link a participant with the information for that person.

128. A. INCORRECT. Beneficence maxim states that above all do no harm
B. INCORRECT. Respect for human dignity includes the right to self
determination and the right to full disclosure.
D. INCORRECT. Code of ethics has three principles: beneficence,
justice and respect.

C. CORRECT. The principle of justice includes the participants right
to fair treatment and their right to privacy which includes
anonymity and confidential procedures.

129. A. INCORRECT. Access for clarification
B. INCORRECT. Debriefing is necessary; it is done to divulge
information that was withheld before the study to clarify issues.
D. INCORRECT. It should not be prejudicial

C. CORRECT. All agreements made are honored, as a feature of fair
treatment to participants in the study, including adherence to the
procedures outlined in advance and payment of any promised
stipends.

130. A. INCORRECT. Included in the informed consent
B. INCORRECT. Included in the informed consent
C. INCORRECT. Included in the informed consent

D. CORRECT. The researcher should provide information on whom
the participants could contact in the event of further questions,
comments or complaints relating research.


































67

REFERENCES USED:

Brunner and Suddarths Medical-Surgical Nursing 10
th
edition
Johnson, G. (1986) Mathematics for Nurses 2
nd
edition
Kozier, B., Erb, G., et. Al. (2004) Fundamentals of Nursing
Pilliteri,A. (2003) Maternal and Child Health Nursing: Care of the
Childbearing and Childbearing Family
Saxton, F., Oneill, N. (1998) Math and Meds for Nurses
Silvestri, L. (2011) Saunders Comprehensive Review for the NCLEX-
RN Examination 5
th
edition






























VMGO

S-ar putea să vă placă și